What is the slope of y-3=-4(x-5)

Answers

Answer 1
Answer:

Answer:

-4

Step-by-step explanation:

y-3=-4(x-5)

y=-4x+20+3

y=-4x+23

y=mx+b where m=slope and b=y-intercept


Related Questions

What is 15/10000 simplified
I need the answers for 23,24,25
How black hole does work.
How much is 300 pennies
Avery tosses a coin 100 times. It lands on heads 60 times and on tails 40 times. What is the experimental probability that it will land on heads

A company manufactures printers.The equation −0.0002x2+0.0563x−0.169=4 can be solved to find the price per printer that the company will need to charge to make an annual profit of approximately 4 million dollars ($4,000,000).

The graphs of y=−0.0002x2+0.0563x−0.169 and y=4 are shown.

What does the graph tell you about the price the company should charge?

Answers

Answer:

It is not possible for the company to earn an annual profit of $4,000,000.

Step-by-step explanation:

it waz right

if right pls mark brainliest

Answer:

get x isolated

Step-by-step explanation:

Hans is drawing triangle ABC on the front board. He wants to draw a triangle with one angle greater than 90°.Which of the following triangles would have an angle that is greater than 90°?

Answers

Final answer:

An obtuse triangle has one angle that is greater than 90 degrees. The sum of all triangle angles is always 180 degrees, hence, to create an obtuse triangle, one angle should be more than 90 degrees and the other two angles together should be less than 90 degrees.

Explanation:

Hans can draw a triangle with one angle greater than 90° by drawing what is known as an obtuse triangle. An obtuse triangle is a type of triangle where one of the angles is greater than 90°. The sum of all angles in any type of triangle always equates to 180°. Therefore, when drawing the triangle, ensure that the sum of the other two angles does not exceed 90°, otherwise, it will not be possible to have an obtuse angle. So Hans should choose the measures of angles wisely.

Learn more about Obtuse Triangle here:

brainly.com/question/2617159

#SPJ12

The triangle that would have an angle greater than 90 degrees would be B. m ∠ A = m ∠ B and m ∠ C =2 ( m ∠ B) and C. m ∠ B + m ∠ C = 82 deg

How to find the triangle ?

m ∠ A = m ∠ B and m ∠ C = 2(m ∠ B):

This implies that angles A and B are equal, and angle C is twice either of them.

Let m ∠ B = x. Then, m ∠ A = x and m ∠ C = 2x.

The sum of angles in a triangle is 180°, so \(3x = 180 \Rightarrow x = 60\).

So, m ∠ C = 120°, which is greater than 90°.

m ∠ B + m ∠ C = 82°:

\[ m \angle A = 180^\circ - 82^\circ = 98^\circ \]

This means that \( m \angle A = 98^\circ \), which is indeed greater than 90°. Therefore, the triangle with the property "m ∠ B + m ∠ C = 82 deg" also has an angle greater than 90°.

m ∠ B = 44° and m ∠ A = m ∠ B + 4°:**

m ∠ A = 44° + 4° = 48°.

The sum of angles A and B = 44° + 48° = 92°.

So, m ∠ C = 180° - 92° = 88°.

None of the angles exceed 90°.

Find out more on triangles at brainly.com/question/36851320

#SPJ2

The full question is:

Hans is drawing triangle ABC on the front board. He wants to draw a triangle with one angle greater than 90". Which of the following triangles would have an angle that is greater than 90°?

length of side AB = length of side CA and m ∠ C = 25 deg

m ∠ A = m ∠ B and m ∠ C =2 ( m ∠ B)

m ∠ B + m ∠ C = 82 deg

m ∠ B = 44 deg and m ∠ A = m ∠ B + 4 deg

WHAT IS THIS SOLUTION OF THE EQUATION? x + 8.63=11.001

Answers

subtract 8.63 from both sides since subtraction is the opposite of addition.

x= 11.001 - 8.63.
x= 2.371
you subtract 8.63 from 11.001 and that equals 2.371. so that is your answer.

What is the number pattern from 10 to 30 to 90 to? and ? to?

Answers

10 - 30- 90-270 - 810
10x3
 30x3
 I am pretty sure that's right

What is 70% of 210. Explain

Answers

Percent\ means\ out\ of\ 100 \n  (70)/(100)*210 \n  \n  ((10)7)/((10)10)*210 \n \n  (7)/(10)*210   \n  \n 7*21 \n \n \boxed {147}
you can set it up as a proportion
is (part)                 %
-------                ------
of( whole)        percent
you fill in the spots so it looks like this. 70% is the percent, and the 210 as the of before it, so by default, the "is" becomes x.
    x          70
------    ------
  210     100  
then you cross multiply, so 100 times x is 100x and 210 times 70 is 14700.
so,        100x=14700
then you have to get x by itself, to cancel out x you do the opposite of the opperation which in this case means to divide, so 100x/100 is x. but what you do to one side, you have to do to another, so 14700/100=147

so X=147, or 147 is 70% of 210
you can also check your answer by multiplying the "of" you get by the percent, and you should get the "is".

What goes into 18 and 42 for equivalent fractions

Answers

6 is a number that goes into both 18 and 42 if that helps, you can divide 18 by 6 and you get 3, then 42 by 6 and get 7, and that is how you simplify that part. I hope that helped :)